what is 54 x 67 ? Plz help!!!!

Answers

Answer 1

Answer:

3618

Step-by-step explanation:


Related Questions

Mr. Calloway has a rectangular yard that is 120 feet wide and 80 feet deep. To keep his dogs on his property, he needs to build a fence. How much fencing does he need?



A. 200 ft

B. 36 ft

C. 400 ft

D. 480 ft

Answers

Answer: what I think the answer is 480 letter D

Step-by-step explanation:

C because 120+120=240 then 80+80=160 160+240=400

The Ramirez family has a circular garden with a diameter of 22 feet. They cover the garden with mulch. The cost of mulch is $1.25 per square foot. To the nearest dollar, how much will it cost to cover the garden with mulch. Show your work.

A.$380 B. $475
C. $1.520 D. 1.900

Answers

Answer:

b) $475

Step-by-step explanation:

if it has a diameter of 22 feet, that means it has a radius of 11 feet

the formula of the area of a circle is πr². so square the radius and you will get 121, multiply that by π and you get 380

Now we know the area is 380 square feet, so multiply that by $1.25 and the answer is $475

b) $475
Step-by-step explanation:
if it has a diameter of 22 feet, that means it has
a radius of 11 feet
the formula of the area of a circle is Tr?. so
square the radius and you will get 121, multiply
that by TI and you get 380
Now we know the area is 380 square feet, so
multiply that by $1.25 and the answer is $475

A farmer wants to increase the area of his rectangular pen but keep the pen a rectangular shape. He decides to add 2 1/2 feet of fencing to the width of the pen, but the length will remain 9 1/4 feet. The new area will be 108 3/4 square feet.

Which equation can be used to determine the original width, w, of the pen?


9 1/4(w+2 1/2)=108 3/4

9 1/4w+2 1/2=108 3/4

2 1/2(w+9 1/4)=108 3/4

2 1/2w+9 1/4=108 3/4

Answers

91/4(w+21/2)=108 3/4

Answer:

914(w+212)=10834

Step-by-step explanation: I took the test

Write the equation of the line that passes through the points (8,−7) and (−9,7). Put your answer in fully reduced point-slope form, unless it is a vertical or horizontal line.

Answers

Answer:

y + 7 = (-14/17)(x - 8)

Step-by-step explanation:

We want the equation of this line in the point-slope form y - k = m(x - h).

We are given two points (8, -7) and (-9, 7), and may arbitrarily choose one of them to serve as the point (h, k) in the point-slope form shown above.  Then the other point is represented by (x, y) in the point slope form shown above:

Let (h, k) be 8, -7) and let (x, y) be (-9, 7).  Then,  y - k = m(x - h) becomes:

7 - (-7) = m(-9 - 8), so that 14 = m(-17), and the slope is thus m = -14/17

The desired equation is then y + 7 = (-14/17)(x - 8) (which is in point-slope form).  

Another way to get this result would be to find m in the usual way:

          rise           14

m = ------------- = --------- = -14/17 (same as before).

           run           -17

Mathematically, why am i so uneducated?

Answers

I getchu the education system is a mess!!

Answer:

I know this is most likely a joke but I'll answer anyways :)

"Mathematically" you might have poor education whether it's bad teachers, bad school, or maybe you're just not working hard enough. Maybe you are educated and you're just comparing yourself against smarter people which might make you seem not very educated. Or maybe you're secretly a super genius prodigy and you're just too lazy too show it. Who knows...

please help ! i'll give brainliest to the first person who gets it right ! :,>

Answers

Answer:

±16

Step-by-step explanation:

the square root of 256 is positive or negative 16! (if it only lets you put one answer, put positive 16).

Answer:

16

Step-by-step explanation:

16*16=256

Answer the following questions.

Answers

Answer:

Thanks for the questions

Thank you so much!!

Giselle pays $279 in advance on her account at the athletic club each time she uses the club $29.99 is deducted from her account. Which answer models her question best

Y = 279(29.99)^x
Y = 279(-29.99)^x
Y = 29.99x - 279
Y = -29.99x + 279

Answers

Answer:

Step-by-step explanation:

B is correct. Your starting amount is 210$ And you're taking away 15$ each time she uses the club,x.

Find the selling price.

Markup ($) = percent markup (%) x original cost ($)

Selling price ($) = original cost ($) + markup ($)


A local gaming store bought a gaming console for $90. The gaming store sold the console, and the percent markup was 162%. What is the selling price?

The selling price is: $_________

Skip to navigation

Answers

Answer:

$145.80

Step-by-step explanation:

The original cost of the product was $90.  There was a 162% markup.  In order to get the selling price, I like to take away 100 percent and simply add original price+(original price*62%).  After taking away 100%, I am left with 62%.  I then multiply the original cost ($90) by the remaining percent (62%), getting 55.8.  Following the steps that I originally wrote, I now add 90 to 55.8, getting 145.8.

Answer:$145.80

Step-by-step explanation:

1/3 + 4/6 +6/18 = ?

pls help
5th grade math

Answers

Exact form: 4/3 decimal form : 1.3 mixed number form : 1 1/3
1/3 + 4/6 + 6/18

1/3•6 = 6/18

4/6•3 = 12/18
add:
6/18+12/18= 18/18= 1
1 + 6/18= 1 6/18 (divide by 6 into simplest form) —> 1 1/3

x = 2 y = 3 z = 4

Evaluate the Expression using the solutions above: yz


7

34

8

12

Answers

It should be 12 but I could be wrong
your answer is 12 because 3x4 is 12

The ratio of athletes in the two teams is 3:2. After 12 athletes move from first team to second, the ratio is 5:4. Find the number of athletes on each team originally.

Answers

Step-by-step explanation:

Let the athletes be 3x and 2x.

Now, By the question we get,

(3x-12)/(2x+12) = 5/4

or, 12x-48= 10x+60

or, 2x = 108

so, x = 54

now

3x= 3×54= 162

2x = 2×54=108

The number of athletes were 162 in first team and 108 in second team originally.

x is the number of athletes on first team originally ( x > 12, x ∈ Nˣ )

y ----------------------------------------second------------------( y ∈ Nˣ )

The ratio of athletes in the two teams is 3:2

⇒      [tex]\frac{x}{y} =\frac{3}{2}[/tex]              (1)

After 12 athletes move from first team to second, the ratio is 5:4

⇒      [tex]\frac{x-12}{y+12} =\frac{5}{4}[/tex]         (2)

(1),(2)  ⇒ [tex]\left \{ {{\frac{x}{y} =\frac{3}{2}} \atop {\frac{x-12}{y+12} =\frac{5}{4}} \right. <=>\left \{ {{x=162} \atop {y=108}} \right.[/tex]

So: The first team originally has 162 numbers and the second team originally has 108 numbers

ok done. Thank to me :>

About 71% of Earth’s surface is covered in water.

Question 1
Part A

If Earth's surface area is about 196,900,000 square miles, about how many square miles are covered in water?

Enter the correct answer in the box.
Part B

About what percent of Earth’s surface is not covered in water? How many square miles is this?

Enter the correct answers in the boxes.

Answers

Answer:

A. 139,799,000 square miles

B. 57,101,000 square miles

Step-by-step explanation:

71% ÷ 100 = 0.71

0.71 x 196,900,000 = 139,799,000

100 - 71 = 29

29 ÷ 100 = 0.29

0.29 x 196,900,000 = 57,101,000

another way; knowledge of A

196,900,000 - 139,799,000

= 57,101,000

You go to a restaurant and order $137.61 worth of food. You decide to tip the waiter 18%. You also have to pay a tax of 6.5% on the original bill. What is the final cost of the meal?

Answers

Answer:

$171.31

Step-by-step explanation:

The final cost of meal is $171.35

What is unitary method?

The unitary method is a technique for solving a problem by first finding the value of a single unit, and then finding the necessary value by multiplying the single unit value.

Given:

price of food = $137 . 61

Tip = 18%

tax = 6.5%

Now, price after giving tip

=137.61 x 0.18

= 24.7698

and, after tax

=137.61 x 0.065

=8.94465

Hence, final cost of meal = 137.61 + 24.7698 + 8.94465 = $171.35

Learn more about unitary method here:

https://brainly.com/question/28276953

#SPJ2

PLEAS HELP




A scuba diver descends in the water at a rate of 2312 feet per minute for 2.6 minutes. He immediately changes course when he sees a large fish and ascends at a rate of 28 feet per minute for 2 minutes.


What is the scuba diver's position relative to sea level after the 4.6 minutes?


Enter your answer as a decimal in the box.


ft

Answers

2312 FPM for 2.6 minutes

2312 x 2.6 = 6011.2

28 FPM for 2 minutes

28 x 2 = 56

6011.2 descend

55 ascend

0-6011.2 = -6011.2

ans + 56 = -5955.2

:)

Answer:

Descends: 2,312 x 2.6 = - 6,011 (when rounding otherwise known as - 6,011.2, this number is negative because he is descending)

Ascends: 28 x 2 = 56

- 6,011 + 56 = -5,955 (in 4.6 minutes)

Step-by-step explanation:

I believe this is the correct answer, I'm not 100% sure though. Please tell me if I'm wrong. Have a wonderful rest of your day or night! ~ ^w^ ~

help with question number 40 please

Answers

y = 1.5x + 6.5
hope this is helpful!
y=1.5+6.5 good luck

In the diagram, line p is parallel to line q. Parallel lines p and q are cut by transversals r and s. At the intersection of lines p, r, and s, clockwise from top left, the angles are blank, 93 degrees, blank, A, C, B. At the intersection of lines q and s, the angles are blank, 60 degrees, blank, blank. At the intersection of r and q, the angles are 27 degrees, blank, blank, blank. Complete the statements based on the diagram. mAngleA = 27° because it is to the 27° angle. The measure of Angle can be found because it is a vertical angle to the 93° angle. The sum of the measures of angles A, B, and C is degrees.

Answers

Answer:

see explanation

Step-by-step explanation:

In the diagram, line p is parallel to line q. Parallel lines p and q are cut by transversals r and s. At the intersection of lines p, r, and s, clockwise from top left, the angles are C, 93 degrees, A, A, C, B. At the intersection of lines q and s, the angles are B, 60 degrees, A, C. At the intersection of r and q, the angles are 27 degree angles, C, A, B.

hope this helps.

Answer: the first answer for dropdown menu1. is (an alternate interior angle)

the second dropdown menu answer is ( C. )

the third dropdown menu answer is ( 180 )

Step-by-step explanation:

mAngleA = 27° because it is  

✔ an alternate interior angle

to the 27° angle.

The measure of Angle

✔ C

can be found because it is a vertical angle to the 93° angle.

The sum of the measures of angles A, B, and C is  

✔ 180

degrees.

PLS ANSWER ASAP!! I WILL GIVE BRAINLIEST TO BEST ANSWER!!
The model shows 1 over 2. Rectangle model divided into two equal sections, first section is labeled one one-half and shaded in. Which of the following correctly models and gives the quotient of 1 over 2 divided by 3 over 10? Rectangle model divided into ten equal sections, first three sections are labeled three-tenths and colored dark, the next two sections are colored light to complete one-half, model equals one and two-thirds. Rectangle model divided into twenty equal sections, first three sections are labeled three-tenths and colored dark, the next seven sections are colored light to complete one-half, model equals three and one-tenth. Rectangle model divided into twenty equal sections, first three sections are labeled three-tenths and colored dark, the next seven sections are colored light to complete one-half, model equals three and one-third. Rectangle model divided into ten equal sections, first section is labeled three-tenths and colored dark, the next four sections are colored light to complete one-half, model equals five.

Answers

Answer:

Ok I guess

Step-by-step explanation:

So first you need understand the question next send a pic maybe I can help U then

what’s the area of this figure? lol i have no idea helppp

Answers

Answer:

285

Step-by-step explanation:

For the top section, we can portion out 20 x 8 = 160.

Next, directly in the middle is a 5 x 5 square = 25.

Finally, we have a right triangle with dimensions 10 x 20 so the area would be 10 x 20 / 2 = 100.

160 + 25 + 100 = 285.

Answer:

take all the numbers and multiply...

Step-by-step explanation:

What is the equation of the line that passes through the point (8,-4) and has a slope of 3?

Answers

In order to solve it you insert the point in slope from

the equation is
Y=Mx+b

Plug it in as

-4=3(8)+b


-4=24+b

Then subtract 24

B=-28

So

The answer is

Y=3x-28
y-y1=m(x-x1)

(8, -4) 8 is x1 and -4 is y1
so y- (-4) = 3 (x -8)
make 4 a positive then y + 4 = 3x - 24
subtract 4 from 24 then y=3x-20

Write an algebraic expression to find the number of seconds in n minutes.


n ÷ 60
60 + n
60 ÷ n
60n

Answers

Answer:

60n

Step-by-step explanation:

The answer will be 60N

8.067 ; hundredths Round each decimal to the place indicated.

Answers

Answer:

8.06

Step-by-step explanation:

Decimals go in the order 10ths hundredths thousandths ten thousandths and so on. In rounding any decimal, one number gets added to the left if the number is 5 or over and the number is discarded if it is lower than 5.

for example if the number was 1.2345      5 would make 4 5 1.235 then 5woould make 3 4 1.24 but if you want to round to a smaller number the other numbers would not go up the numbers would just be discarded.

Show me how to do this on paper i need it right now

Answers

Answer:

Step-by-step explanation:

32 x 10 = 320

32 x 4 = 128

320 + 128 = 448

(0,-5), (1,-4), (2, -3), (3, -2), (4, -1), (5,0) is this a relation

Answers

The answer is yes. You add one every time.

What is the distance from City A to City C?



A. 12 km
B. 18 km
C. 20 km
D. 30 km

Answers

Answer:

30

Step-by-step explanation:

the whole thing is 47 so you take away 17 because thats the starting point of c to d so then you have ur answer hope this helped :))))))))))

Answer:

It is D

Step-by-step explanation:

You just do this

[tex]47 - 17 = 30[/tex]

A line passes through (3, 7) and (6, 9). Which equation best represents the line?
y = 3/2x + 5

y = 2/3x + 5

y = 3x + 2

y = 2/3x + 2

Answers

Answer:

y=2/3x + 5

Step-by-step explanation:

y=2/3x + 5
is the answer

What is the correct order of statements and reasons to complete the proof? help please

Answers

The answer will be A due to them being on opposite sides with the triangle having adjacent measures so both of those angles given provide the same measure
The answer would be A

22/39 of 3/11 plz help asap :)

Answers

Answer:

2/13

Step-by-step explanation:

To find a fraction of a fraction, you just multiply the two.

So 22/39 x 3/11 is 22x39 over 3/11

or 66/429.

Simplified would be 22/39.

The value of 22 / 39 of 3 / 11 on simplification is 6 / 39.

What is multiplication?

The fundamental concept of making the same number of additions repeatedly is represented by the action of multiplication. The results of multiplying two or more integers are known as the products, and the factors that are used in the multiplication are referred to as the factors.

Given:

22 / 39 of 3 / 11

As we know of means multiplication then,

22 / 39 × 3 / 11

(22 × 3) / (39 × 11)

66 / 429

This can be simplified as,

6 / 39

To know more about multiplication:

https://brainly.com/question/1135170

#SPJ2

A diver descends to a depth of –25.6 feet relative to sea level. How many feet must the diver ascend to reach sea level?
0.6 feet
25 feet
25.6 feet
256 feet

Answers

The answer is 25.6 feet.

Hope this helps

when keisha installed a fence along the 200 feet perimeter of her rectangular back yard she left and opening for GATE in diagram below she used X to represent the length of the gate

Answers

Perimeter of her rectangular back yard is: 200 ft

⇒ 2(30 + 60 + x) = 200

⇒ 30 + 60 + x = 200/2 = 100

⇒ 90 + x = 100

⇒ x = 100 - 90 = 10

ANSWER: A.10

Ok done. Thank to me :>

Other Questions
please help me and solve it having appeared in the film love and basketball, actress gabrielle union later married what nba player? One leg of an isosceles right triangle measures 5 inches. Rounded to the nearest tenth, what is the approximate length of the hypotenuse?2.5 inches5.0 inches7.1 inches9.8 inchesPLZ ANSWER ASAP what effect does the tone of the excerpt have on the reader? it fosters a belief that the narrator is unreliable. it produces a contradictory urge to stop reading and to continue. it inspires confidence that everything will work out fine in the end. it encourages surprising delight in blood and gore. Sue has 100 dimes and quarters. If the total value of the coins is &21.40, how many of each kind of coin does she have? yesterday, john had $15. In 7 days, He would have 23. How do you divide fractions? Can someone please explain step by step to get marked as brainliest! Select the answers that best match for that political party's beliefs. What is the full meaning ofSOCRATES10k40 secondsStarts now PLEASE QUICK HELP ME!! ill give brainlist HOME WORK DUE SOON PLZ HELP!!!!!what is the slope of this graph? A worker at a clothing store earns a 20% commission on a sale of $85.00. How much is the commission The __________ step of the Study Cycle involves reflecting on your mastery of understanding. In this step, you should ask yourself if you understand the material enough to teach it to someone else. PreviewAttendReviewStudyCheck A jar contains black marbles and green marbles. Sixty percent of the marbles in the jar are green. A number generator simulates randomly selecting 10 marbles from the jar. The number generator is used 12 times and the number of green marbles in each trial is shown in the dot plot.Which description is correct about the number generator being fair or not?The number generator is not fair. The dot plot shows 80% of the marbles drawn are green.The number generator is fair. The dot plot shows a distribution that is skewed right.The number generator is not fair. In most of the experiments, considerably less than 60% of the selected marbles are green.The number generator is fair. It shows that 60% of the marbles selected are green most of the time. What is accomplished by the following code snippet, assuming all variables had beenpopulated?amount_owed += duration rateO Amount owed is set to duration times rate.O Amount owed adds duration times rate to it.O Amount owed is displayed to a user as duration rate.O This line would return a syntax error. EspaiA house on the market was valued at $224,000. After several years, the value decreased by 9%. By how much did the house's value decrease in dollars? Whatis the current value of the house?Decrease in value:Current house value: i never learned fractions plz help Hey this is passed due but i want to get my stuff done the question is, What did John Adams do to convince delegates to sign the Declaration of Independence? look at the graph. which equation represents the graph? isotopes of an element contain different numbers of